Representations of lorentz group and transformations IN DETAIL

Phi,$$where $\theta^i$ and $\beta^i$ are the infinitesimal angles and boosts, respectively.Using the fact that $J=\sigma/2$ in the spin-1/2 representation, we can write the transformation explicitly for the $(1/2,0)$ and $(0,1/2)$ representations as$$\psi_R \rightarrow \left(i\theta^i\frac{\sigma}{2} \pm \beta^i\frac{\sigma}{2}\right)\psi_R,$$where $\psi_R$ is a spinor. This transformation is precisely the same as the one for Weyl spinors, which are a reduced representation of the Lorentz group with spin $(
  • #1
Onamor
78
0
From Peskin and Schroeder:
The finite-dimentional representations of the rotation group correspond precisely to the allowed values for the angular momentum: integers or half integers.
From the Lorentz commutation relations:
[itex]\left[J^{\mu \nu},J^{\rho \sigma}\right]=i \left(g^{\nu \rho}J^{\mu \sigma}-g^{\mu \rho}J^{\nu \sigma}-g^{\nu \sigma}J^{\mu \rho}+g^{\mu \sigma}J^{\mu \rho}\right)[/itex]
we can define rotations [itex]L^{i}=\frac{1}{2}\epsilon^{ijk}J^{jk}[/itex] and boosts [itex]K^{i}=J^{0i}[/itex]. An infinitesimal Lorentz transformation can then be written [itex]\boldsymbol{\Phi}\rightarrow\left(1-i \boldsymbol{\theta}.\boldsymbol{L}-i \boldsymbol{\beta}.\boldsymbol{K}\right) \boldsymbol{\Phi}[/itex].
The combinations [itex]\boldsymbol{J_{+}}=\frac{1}{2}\left(\boldsymbol{L}+i\boldsymbol{K}\right)[/itex] and [itex]\boldsymbol{J_{-}}=\frac{1}{2}\left(\boldsymbol{L}-i\boldsymbol{K}\right)[/itex] commute and satisfy the commutation relations of angular momentum.
This implies that all finite dimensional representations of the Lorentz group correspond to pairs of integers or half integers, [itex]\left(j_{+},j_{-}\right)[/itex], corresponding to pairs of representations of the rotation group. Using the fact that [itex]\boldsymbol{J}=\boldsymbol{\sigma}/2[/itex] in the spin-1/2 representation of angular momentum, write explicitly the transformation according to the [itex]\left(\frac{1}{2},0\right)[/itex] and [itex]\left(0,\frac{1}{2}\right)[/itex] representations of the Lorentz group. You should find these correspond precisely to the transformations for the Weyl spinors:
[itex]\psi_{R,L}\rightarrow \left(i \boldsymbol{\theta}\!\frac{\boldsymbol{\sigma}}{2}\pm\boldsymbol{\beta}\!\frac{\boldsymbol{\sigma}}{2}\right)\psi_{R}[/itex].I don't know how to answer the question as I don't understand some of the preamble, so I will try and motive the (colour-coded) sentances in detail, and I'd be very grateful if someone could tell me where my reasoning is incorrect:

The "representations of the Lorentz group" means the [itex]\boldsymbol{J_{+}}[/itex] and [itex]\boldsymbol{J_{-}}[/itex], as they satisfy the Lorentz commutation relations. Where it says "correspond to" pairs of integers or half integers, it means these representations (ie matrices) "produce the eigenvalues" denoted [itex]\left(j_{+},j_{-}\right)[/itex]. But why are they in pairs? Doesn't acting with one [itex]\boldsymbol{J_{+}}[/itex] or [itex]\boldsymbol{J_{-}}[/itex] produce one eigenvalue; [itex]j_{+}[/itex] or [itex]j_{-}[/itex]?
The [itex]\left(j_{+},j_{-}\right)[/itex] correspond to representations of the rotation group means that there are some pairs of representations of the rotations group that will give the same [itex]\left(j_{+},j_{-}\right)[/itex] as eigenvalues. By "pairs of representations" they mean one representations will give [itex]j_{+}[/itex] and the other will give [itex]j_{-}[/itex].
The [itex]\left(\frac{1}{2},0\right)[/itex] and [itex]\left(0,\frac{1}{2}\right)[/itex] are spinors, ie a "reduced" representation of the Lorentz group. The designation of half integers seems somewhat arbitrary to me, I don't know what the significance of these numbers is...

To do the question I'm guessing I have to somehow relate [itex]\boldsymbol{J_{+}}[/itex] and [itex]\boldsymbol{J_{-}}[/itex] to [itex]\boldsymbol{J}[/itex] and replace [itex]\boldsymbol{\Phi}[/itex] in the infinitesimal Lorentz transformation with the spinors.
But as shown there is some understanding missing, and I wouldn't really know what I'm doing.

Sorry for the excruciating detail, and many thanks to anyone that can help. If I haven't been clear or you think I should split this up into more than one post please let me know.
 
Physics news on Phys.org
  • #2
A:In a finite-dimensional representation of the Lorentz group, the generators $J^{\mu\nu}$ are represented by matrices. Each matrix will have a set of eigenvalues, which correspond to the allowed values for angular momentum. Since these generators satisfy the Lorentz commutation relations, they can be written as the linear combination of rotations and boosts given by$$J^{\mu\nu} = \frac{1}{2}\epsilon^{ijk} L^i J^{jk} + K^i J^{0i},$$where $L^i$ and $K^i$ are the generators of rotations and boosts, respectively.The eigenvalues of the $J^{\mu\nu}$ must obey the same commutation relations, so they can be written as a linear combination of eigenvalues of the $L^i$ and $K^i$ as$$j^{\mu\nu} = \frac{1}{2}\left(j_+^i L^i + j_-^i K^i\right).$$The eigenvalues of the $L^i$ and $K^i$ are integers or half-integers, corresponding to the allowed values of angular momentum. So the eigenvalues of $J^{\mu\nu}$ must also be integers or half-integers. Furthermore, since they must obey the same commutation relations, the eigenvalues must come in pairs, with one from $L^i$ and one from $K^i$. Thus, in a finite-dimensional representation of the Lorentz group, the allowed eigenvalues of $J^{\mu\nu}$ are given by pairs of integers or half-integers $(j_+,j_-)$.Now, for the spin-1/2 representation of the Lorentz group, the generator $J^{\mu\nu}$ is related to the Pauli matrices as $J^{\mu\nu}=\frac{1}{2}\sigma^{\mu\nu}$. An infinitesimal Lorentz transformation can then be written as$$\Phi \rightarrow \left(1 - i\theta^i L^i - i\beta^i K^i\right
 

Related to Representations of lorentz group and transformations IN DETAIL

1. What is the Lorentz group and why is it important in physics?

The Lorentz group is a mathematical group that describes the transformations between different frames of reference in special relativity. It is important in physics because it allows us to understand the properties of space and time in a consistent way, and it is essential for making accurate predictions in high-energy and high-speed phenomena.

2. What are the elements of the Lorentz group and how do they relate to each other?

The elements of the Lorentz group are Lorentz transformations, which include boosts and rotations. These transformations relate different frames of reference and can be combined to create more complex transformations. Boosts describe how an object's position and time coordinates change when moving at a constant velocity, while rotations describe how its spatial orientation changes.

3. How do Lorentz transformations differ from Galilean transformations?

Lorentz transformations take into account the effects of special relativity, such as time dilation and length contraction, which are not present in Galilean transformations. Additionally, Lorentz transformations are non-commutative, meaning the order in which they are applied matters, while Galilean transformations are commutative.

4. How are Lorentz transformations represented mathematically?

Lorentz transformations can be represented by 4x4 matrices, which consist of elements that relate the coordinates of one frame of reference to another. These matrices can be written in terms of the Lorentz group's generators, which are mathematical operators that generate the transformations.

5. Can the Lorentz group be extended to include other transformations?

Yes, the Lorentz group can be extended to include additional transformations, such as translations and scale transformations, to form the Poincaré group. This extended group is important in general relativity, where it describes the symmetries of spacetime and plays a crucial role in Einstein's field equations.

Similar threads

Replies
5
Views
2K
  • Advanced Physics Homework Help
Replies
2
Views
869
  • Advanced Physics Homework Help
Replies
5
Views
2K
  • Advanced Physics Homework Help
Replies
1
Views
1K
  • Advanced Physics Homework Help
Replies
1
Views
1K
  • Advanced Physics Homework Help
Replies
2
Views
575
  • Advanced Physics Homework Help
Replies
8
Views
861
  • Advanced Physics Homework Help
Replies
3
Views
1K
Replies
4
Views
2K
  • Special and General Relativity
Replies
7
Views
1K
Back
Top